« first day (1981 days earlier)      last day (2942 days later) » 

7:00 PM
@0celo7 wavefunction $\neq$ observable
i.e. it is not a physical quantity
 
elements 1,3 and 5 are definitely in $B$ so it's just a matter of if addition between two elements is inside of $B$
 
How does the Schroedinger equation work if it's not at least C2
 
@JohnRennie No, if the flag was truly justified then logically action should have been taken which would be equivalent to an actually-validated flag. That is, the post should have been deleted and the user should have been kicked.
 
2 mins ago, by ACuriousMind
Oh, for god's sake, stop trolling.
 
Fuck, I'm not trolling
 
7:02 PM
yes you are
Schrödinger equation is perfectly defined on $L^2$
 
Better ban me now mods
I'm going to get banned anyway
 
and you know that
 
How would I possibly know that
 
Aug 19 '15 at 2:10, by 0celo7
@ACuriousMind yes but he does not talk about your stone theorem
 
(sorry guys, phone)
 
7:04 PM
@yuggib what?
I don't even know what that is
 
(catching up now)
 
Aug 20 '15 at 2:35, by 0celo7
@ACuriousMind I want to say Stone's theorem has something to do with it.
 
@Obliv Reading what the problem says, what do you think B is and why?
 
you knew 8 months ago
 
What part of I CANT REMEMBER ANYTHING ANYMORE don't you get
 
7:07 PM
@barrycarter I think $B$ is the set of matrices that are commutable in multiplication with $M$
 
@Obliv Right. And what matrices are in that set?
@Obliv I think the list you provided earlier was incomplete.
@Obliv If X and Y commute with M, then MX = XM and MY = YM, right?
 
oh I didn't check all of them 1 sec
 
@Obliv You can actually prove it using the distributive property without necessarily enumerating the set.
 
oh does $\begin{bmatrix}1&1\\1&1\end{bmatrix}$ work too?
 
@yuggib Do you really not believe me
 
7:09 PM
I think so, yes.
 
this doesn't help since some of these added to each other are still not in $B$ tho
 
I don't see why a PDE would be defined on a discontinuous function
 
Actually, why not solve that problem analytically?
@Obliv I don't think it says anywhere that those are the only matrices in B.
 
@0celo7 Hint: in the sense of distributions
 
@Obliv Multiply M by an arbitrary matrix in both directions, and see when you get equality.
 
7:10 PM
I don't know what that means
 
Ocelot, PDE or PDF?
 
@0celo7 do you know what a distribution is?
 
No
 
what is the Dirac's delta?
 
The Delta Dirac distribution has a well defined CDF, but no well-defined PDF.
 
7:11 PM
@yuggib a distribution
 
@barrycarter so any matrix that satisfies commutativity when multiplied with $M$ is in $B$. that doesn't change the fact that $P,Q \in B$ must mean $P+Q \in B$
 
how it acts?
 
Which it doesn't. for many matrices.
 
Linear functional on a function space?
 
and on what?
@0celo7 exactly
 
7:12 PM
@Obliv Well... (P+Q) x M = P x M + Q x M, right?
 
For anyone who hasn't done so recently, this may be worth a read.
42
A: A guide to moderating chat

bluefeetWhat tools are available to regular chat users? You, as a member of the community, have the ability help keep chat a place that is welcoming for everyone. There might be a variety of other users, including moderators and room owners, in chat with you. Users can be distinguished by the way thei...

3
> Remember: Only flag things that are truly inappropriate or offensive.

When you flag a message, you draw the attention of every mod and 10 k user currently on the chat network. This means you should only be flagging things that they’re going to see as offensive as well, which includes:

Personal attacks (characterized by you, e.g., “You’re a moron.”)
Anything that shocks the conscience (yes, this is subjective — let your conscience be your guide.)
 
@barrycarter yes
 
@Obliv And P x M = M x P and Q x M = M x Q since both commute with M, right?
 
and derivatives of distributions can be defined, by the corresponding action on test functions
 
@barrycarter so you're saying because (P+Q)M = (Q+P)M that those quantities are in $B$?
 
7:14 PM
@Obliv If you meant Q + P, yes.
 
yes
again, very tired q.q
 
@Obliv Yes, if P and Q are in B, then P + Q must also be in B.
 
It's also worth noting that spam/offensive is not your only flag option. You can also "flag for moderator". The process for that includes a short form where you can provide some amount of context and/or describe what action is really required for the message. And that only goes to moderators - not mods+10k.
 
so if an equation is satisfied in a distributional sense, it means that it is satisfied by a distribution (when acting on test functions)
 
@Obliv By the distributive property of multiplication/addition on matrices.
 
7:15 PM
@barrycarter that just means that matrices P and Q are in B. That doesn't mean the sum of P and Q are in $B$.
@barrycarter wait no.. that does mean the sum exists.
 
@Obliv OK, (P + Q) x M = P x M + Q x M, right?
 
that makes no sense WTF.
 
@Obliv Dude, get some sleep.
 
@yuggib I know you want me to say Schwarz space but I can't prove it.
So why would I know any of this?
I'm an engineer
 
@barrycarter are you so sure that the distributive property of (P+Q)M is commutative?
I must prove this.
 
7:18 PM
@James Hm. I think that answers a question I just had too.
Thanks.
 
@Obliv I'm saying you can write it out step by step, yes.
 
@0celo7 you always try very hard to convince yourself and the others that you know less than what you actually know
and knowing that a result holds is not equivalent to knowing all the proofs involved
 
Because people always overestimate me
WHAT
I must move to one of these cities
 
Lifting of the day is done B)
My arms
I've got a whole lot of fitness to do to take care of that dishful of chicken grease I ate yesterday
 
7:23 PM
not too expensive
 
O'neil
 
Too expensive
Will wait until it gets cheaper again
 
@barrycarter how rigorous does a proof have to be? If $P,Q \in B$ then $PQ \in B$. It follows that $PQ(M) = QP(M)$. Therefore, $PQ \in B$. Is that not rigorous enough?
 
@Obliv Wait, how do you know that P, Q in B means that PQ is in B... or is that what you're trying to prove?
 
7:29 PM
that's what I'm proving, yes.
 
@Obliv Why does PQ(M) = QP(M) ... P and Q are only guaranteed to commute with M, not each other.
 
but they're all matrices
 
@Obliv And?
 
isn't matrix multiplication commutative?
 
@Obliv o.O
 
7:30 PM
oh it's not always. DAMN.
 
@Obliv If it were, the problem would be trivial. B would be all 2 x 2 matrices.
 
can I prove the statement with a constraint?
or does it have to be true for all $P,Q \in B$ do you think/
 
@Obliv All P, Q in B, obviously.
 
ok I have an idea i'll brb.
 
@Obliv You're trying to show the B is closed with respect to addition and multiplication.
 
7:41 PM
hey @barrycarter in $P(QM)$ QM is commutative so it is the same as if it were $P(MQ)$ right? And $(PM)$ is commutative so it would be the same as $M(PQ)$ so that proves commutativity for $(PQ)$ on $M$ doesn't it?
 
@Obliv Very good, yes. You're using associativity to prove commutativity.... wait.
 
i'm using the commutativity of the parts to prove the commutativity for the quantity.
 
(PQ)M = M(PQ)... yes, taht wrks
Well, yes, but you need P(MQ) = (PM)Q at one point
 
yeah matrix multiplication is associative right?...please god just say yes
 
yes :)
I was just pointing out that you were using it implicitly
 
7:47 PM
psh yeah i knew that
 
@yuggib I never use a result that I don't know how to prove.
 
Hey guys, in magnetism i encounter phrase like "Time varying magnetic field" , "increasing or decreasing magnetic field". Can some one please explain their meaning and significance? A small help can change my life :P
 
They mean exactly what they say. A "time varying magnetic field" is a magnetic field that changes with time. What about that is unclear to you?
 
whats is need?
 
What?
 
7:58 PM
why time varying? does it gives the direction?
i mean why do we have to vary the magnetic field?
 
We don't "have to" vary the field - it varies all by itself (in most situations because the current that generates it varies).
 
You mean to say Current varies so Magnetic field varies?
So will it do any change if its a constant Magnetic field and not a time varying?
 
@0celo7 yes you do
 
@JimmyKudo Any change to what?
 
I am particularly concerned about numerical not experiments
 
8:02 PM
I don't really get what your issue is, what situation are you looking at, and what troubles you about the field varying in time?
 
To every aspect related to the magnetic field
 
Of course a magnetic field that doesn't change with time is different from one that does change in time!
 
I am just asking the significance of the "time varying" magnetic field
Can u say an example which produce time varying magnetic field please
 
@JimmyKudo Alternating current in a wire.
 
ok now I understood, thanx
For DC its constant right?
 
8:04 PM
Yep
 
Thanx Acuriousman :)
 
@yuggib Proof?
 
nya nya nya
 
@Acuriousman thank you for all of your help too :)
 
I'm a mind, not a man ;)
3
 
8:05 PM
PROOF
you're an a.i
 
i second that
 
U are bot? I see you always online :P
 
he knows too much
 
Damn, you've finally unmasked me :O
 
8:09 PM
@ACuriousMind Are you watching the eagles?
 
@BernardMeurer Should I?
 
Do they something except being eagles?
 
Both eagles and both babies on the nest
Idk
 
I keep thinking you're talking about football then I realize you're both not from the U.S. lol @BernardMeurer
 
8:10 PM
It's weirdly entertaining
THEY ARE FEEDING
 
...they're named "Liberty" and "Justice"?
 
@ACuriousMind Hell yeah
 
any one watches The Flash?
arent those eagles scared of the cameras and all
 
How can I determine that $f: \mathbb{Q} \to \mathbb{Z}$ defined by $f(a/b) = a$ is well-defined? I feel like $f(.5)$ and $f(1/2)$ will both return $1$, no?
 
Meh, they're just being eagles
Also I keep getting an ad every minute
 
8:13 PM
use adblock
 
Use adblock
 
NoScript perfectly kills all other ads, no need to install adblock
 
@Obliv Hmmm, can you write the same rational number in more than one way?
 
@Obliv It's not well-defined.
 
great. I'll just write : because ACM told me it is not well-defined.
 
8:14 PM
@0celo7 what is the proof of Stokes theorem?
 
@Obliv Oh, did you mean "whether" and not "that"?
 
@ACuriousMind Good catch
@Obliv Are we getting a little snippy with ACM?
 
lol
just foolin, m8s
how did you come to that conclusion, though. @acuriousmind
 
To show it's not well-defined, do what @barrycarter said.
 
wow that was obvious.
can't believe I didn't catch that >:(
 
8:16 PM
@Obliv I can believe it.
 
not helping >_>
 
@Obliv Have you looked at the chat history? You should get some sleep if you're tired.
@Obliv At one point, you claimed the elements of B were in B because I had confirmed it for you.
 
Yeah I kind of assumed $B$ was a closed set because it said choose the elements that belong to $B$. VERY VERY dumb assumption.
so I thought maybe the elements I chose to be in $B$ were not actually in $B$ that's what I meant
 
@Obliv Oh, OK, that makes more sense.
 
I take the bloody physics gre again on the 16th. . . me so scared :O
 
8:27 PM
good luck man :P @kevinTahN.
 
:28843115 first, you need to know how to integrate (differential forms) ;-P
 
@yuggib I do actually know how to prove that
I wrote the full proof once in my own words, it's in my private notes.
 
@Obliv thanks man!! :D
 
Ocelot, and Fermat's Last Theorem too?
 
@barrycarter Nah, margin was too small.
 
8:31 PM
@0celo7 so you know how to define integration of differential forms as well
with all the proofs of the relevant properties
 
Of course, pullback onto Rn using charts
Partition of unity
All that good stuff.
I don't know why are trying to knock me
IF there is something I can do, it's geometry
 
@0celo7 just accidentally checked out your blog. cool beans!!
 
I am not trying to knock you, you asked me for a proof that you use results of which you do not know the proof
and I am not sure that you know all the relevant proofs about integrals (in $\mathbb{R}^n$ if you used charts) that you need
it is not something bad, it is just that there are a lot of things that people use without knowing the proof
 
@yuggib The only thing I can't do is the Fubini theorem.
You need that to prove the Stokes theorem.
But I do know """most""" of the proof.
 
as I told you, it is not a bad thing to use a result without knowing all the underlying proofs
but it is unfair to say "I do not use results of which I do not know the proof"
maybe unfair is not the correct word...exaggerated is better perhaps
 
8:43 PM
@DanielSank I summon you!
 
I strongly believe you just need to know what a set is and how to draw commutative diagrams and you will understand 30% of math, which is enough to pretend :D
 
Diagram chasing proofs are really those proofs you do once and pray never to have to do again
Snake lemma, horseshoe lemma, etc. They're not really difficult, but they're so lengthy.
And most people really do prove them by diagram chasing without having seen the proof of that embedding theorem that shows that that suffices for categories other than abelian groups/modules.
 
interesting, I have only chased diagrams for very basic proofs recently ~ a few weeks now. Mostly about little things in hopf algebra. It was amazing. All I had to do was draw a diagram and make statements concerning where the arrow were pointing and composing them. It was amazing!!
Also I realized if I drew a venn diagram things became very clear! I have seen this in algebra, probability theory and I even used it to see what some of the statements in rosenlicht's into to real analysis meant
If someone had told me this a few years ago, I would have been kinda smart by now
I mean, I have no formal training as a mathematician or anything lol
Also i noticed from the intro to analysis text that you can proof anything by using supremum and infinimum concepts
. . . . . .most of the time
 
8:59 PM
@BernardMeurer What is thy bidding, my master?
 
0
Q: Is it okay to bump questions?

knzhouMost of my questions are about quantum field theory. They tend to get extremely few views and a handful of positive votes before sinking off the bottom of the active list. My non-QFT questions are pretty well-received, so I'd like to think I'm not asking totally trivial questions, but almost all ...

 
Well I just sat in the most boring talk of all time
What I thought it was going to be on: power series
 
is the reflexivity of a relation (say $\sim$) guaranteed in sets? Like {$a,b,c\}$ and $a\sim a$ is guaranteed?
 
What it was on: combinatorics and graph theory
 
9:01 PM
sounds like the later should be interesting
 
@Obliv reflexivity is by definition required of all eq. rels
 
let me reword
 
@Slereah Are you around
 
@0celo7 how about now? If it's just a relation and not equivalence relation
 
@Obliv no, is $1>1$?
 
9:03 PM
hmm sets. . .
what is "reflexivity" as used above with regards to sets
 
@ACuriousMind Do you have a "cooler" definition of $TM$ than $TM=\sqcup T_pM$?
 
In mathematics, a reflexive relation is a binary relation on a set for which every element is related to itself. In other words, a relation ~ on a set S is reflexive when x ~ x holds true for every x in S, formally: when ∀x∈S: x~x holds. An example of a reflexive relation is the relation "is equal to" on the set of real numbers, since every real number is equal to itself. A reflexive relation is said to have the reflexive property or is said to possess reflexivity. == Related terms == A relation that is irreflexive, or anti-reflexive, is a binary relation on a set where no element is related to...
 
> For the proof, see Steenrod
AHHHHHHH
@yuggib can you prove Stokes' theorem?
 
@0celo7 no
 
@yuggib why not
 
9:15 PM
I thought people accidentally learned to do this when they learned about maxwell's equations
differential and integral forms
 
@0celo7 No. Sadly, it's not the étalé space of the sheaf of vector fields.
 
@DanielSank I need your help
Regarding UCSD
Are you aware of any QC over there?
I recall reading something about a Fermionic QC
but I don't even know what that is
 
@ACuriousMind ...What?
 
9:29 PM
@ACuriousMind Are you going to get the F3 DLC?
 
@0celo7 never looked into it
 
@ACuriousMind Are you there?
 
@0celo7 no...
don't know it
 
@BernardMeurer I honestly don't know.
 
@DanielSank What's a fermionic QC though?
 
9:37 PM
0
Q: Are physical functions always differentiable

TakirionI know that physicist usually don't really think too much about differentiabillity of functions. Usually there are at most finite many points where functions aren't differentiable and if there are infinite many of them they are all isolated. On the other hand you sometimes see examples (especiall...

related to before...
 
9:49 PM
@DanielSank csqc.ucsb.edu
These guys are over there
 
@BernardMeurer Somewhat, what's up?
@0celo7 Nope
 
@ACuriousMind What are Bell inequalities?
 
@BernardMeurer Inequalities discovered by Bell that many people make a large fuzz about
 
@BernardMeurer Did you try googling
@ACuriousMind wtf if I asked that you would tell me to google
 
Also, that^^
 
9:58 PM
@0celo7 Yes, it didn't yield me a good answer
@ACuriousMind Got it
 
@0celo7 Note that I didn't actually tell him anything of substance :P Also, you're not my children, I'm not required to treat you all equally
 
oO
You openly admit you treat me like less than @BernardMeurer
 
@0celo7 The fact that I don't ask him a 100 things a day might be why
 
@0celo7 I treat you differently. I'll refrain from value judgements.
 
I haven't asked him anything lately
 
10:00 PM
53 mins ago, by 0celo7
@ACuriousMind Do you have a "cooler" definition of $TM$ than $TM=\sqcup T_pM$?
 
cough
 
@0celo7 I'm clearly the favorite child
 
@BernardMeurer Honestly, I never cared much about those inequalities, but then, I never had issues with accepting quantum mechanics.
 
@ACuriousMind Meh, I trust you people way too much to not accept QM
 
@BernardMeurer apparently :(
 
10:02 PM
::hands @ArtOfCode cough drop::
 
cheers
 
Trying to prove $\sim$ is an equivalence relation. $a\sim b$ if and only if $f(a) = b$. There exists an inverse such that $g(b) = a$. does that mean I can say $b \sim a$ and that $\sim$ is symmetric?
 
@ACuriousMind that's not even a question
@ACuriousMind Have you heard anything about Finsler geometry?
 
Something about manifolds with path length metrics but no actual metric tensor or somesuch, but I know nothing specific.
 
@ACuriousMind you're so smart
@Slereah Got the Asian edition of KN coming, dat classic tho
It's gonna take a month
 
10:27 PM
@Slereah hopefully it's in English
 
@BernardMeurer Not a clue.
 
Ged demmit dan
@DanielSank Why does SB have to be far from LA and SF
 
@BernardMeurer You mean these groups at UCSD, right?
@BernardMeurer You know... because.
 
I found this quantum.ucsd.edu
But the latter seems dead
and the former is the fermionic QC I was talking about
Can't wait to hear back from IQC
Holy
 
Chill
 
10:42 PM
I need to answer UCSD before the results for the IQC come out
What do I do
 
And don't eat any coffee beans
 
@ACuriousMind I already finished my package of those, they're so good man, @ChrisWhite was right
 
dammit I was doing something
then I was interrupted
and have no clue what I was doing
 
@0celo7 What do I do?
 
About?
I don't remember our conversations
 
10:48 PM
The application decisions for the IQC are released after my deadline for accepting UCSD's admission offer
 
What's the penalty for accepting and then not going?
 
Eternal shame, probably
 
Dunno, how do I google for that?
Does that penalty have a name?
 
@ACuriousMind Don't you talk to me about eternal shame
Mr. "let's shame 0celo7 for asking questions"
@BernardMeurer Call and ask?
 
HA
NO PENALTY
"Yes, you may decline after accepting admission. The application site will not allow you to change your decision from “Accept” to “Decline”, so you must contact your department directly to inform them of the change in your decision."
 
10:54 PM
@BernardMeurer Um...I may be wrong, but that's for graduate students, isn't it?
you're applying for undergrad, right?
 
@ACuriousMind You're right.
 
@ACuriousMind Crap
 
@BernardMeurer Call them tomorrow.
 
Shit I don't want to call them, they'll get angry that I'm considering other schools
What if I get a bitchy admissions officer and they're like "Oh, you're considering not taking it? Let me make your life easier and deny you"
 
@BernardMeurer I'm going to go out on a limb and say they won't. Not if they want to stop you from going to those other schools.
 
10:57 PM
I'll call with a fake name
I'll be Björn Merkwürdigliebe
 
They'll be real happy if they find out you pretended to be someone else, yeah.
 
Shit dude I'm going to have a stroke
 
@BernardMeurer My love for Bajoran is pretty strange.
 
That's a cool name right?
 
No.
 
11:01 PM
@ACuriousMind Do you think I could be a Björn?
Will you welcome me into the clan?
 
I like Björn Kleinschwanz.
 
11:39 PM
Well this is odd... recent answer posted to a 2-year-old question and accepted? Also, it sounds like nonsense
-6
A: What is the significance of Planck force?

ZSDThe maximal possible force in universe is something that scientists missed to consider, and if they would consider it, then everything what they have done so far would not be worth a dime, ranging from "point-like particles" and up to the "Big Bang". Point-like particle (i.e. photon), or any othe...

 
@DavidZ It probably got accepted because it has nice TeXed formulae :P Every answer on that page except yours is nonsense.
@BernardMeurer You're either born a Björn or not. Sorry, I don't make the rules!
 
@ACuriousMind Dang it
 
@DavidZ: I won't do it because we don't delete wrong answers, but I see a delete button there. Can a negatively scored accepted answer be deleted by 20k delete votes?
 
@ACuriousMind I'm not sure
If you see the button there, you should use it as appropriate
While we don't delete answers for being wrong, we do delete answers that don't represent serious attempts to answer the question using relevant knowledge.
(apparently)
 
If we're ever tired of discussing homework, we could do that policy next ;)
 
11:52 PM
That's on you ;-) I have my hands full with homework and stuff
 
How's the bot coming along?
 
Actually, I wouldn't care much about leaving wrong downvoted answers on the site if they didn't act as net rep generators
That user derived 21 rep points from posting that nonsense, although the post sits at -7!
Which isn't a concern if they are drive-by posters, but it is if they're persistent.
 
It's a little bit of a concern
 
@ACuriousMind Can you have more than two particles in an entanglement?
 
3
Q: three-particle quantum entanglement

user27515So I know that two particles can be entangled in a quantum way, but is it possible that more than two particles be entangled in a quantum way? Most descriptions provide with two-particles cases, so I wonder. (It's hard to think of three particles entangled in spin, or so.)

 
11:59 PM
Use your Google-fu :P
 
Holy
Imagine a 10 qubit entanglement
 

« first day (1981 days earlier)      last day (2942 days later) »